Está en la página 1de 8

E SCUELA P OLITÉCNICA N ACIONAL

Á LGEBRA L INEAL • H OJA DE EJERCICIOS NO . 09

Semestre 2019-A Departamento de Formación Básica

1. Sea V un espacio vectorial y W1 y W2 dos subespacios de este Probar que:

a) W1 + W2 es un subespacio de V.
b) W1 y W2 están contenidos en W1 + W2
c) W1 + W2 es el menor subespacio de V que contiene a W1 y a W2 , esto es, W1 + W2 = gen(W1 , W2 ).
d) si W1 ⊆ W2 , entonces W1 + W2 = W2
e) si W2 ⊆ W1 , entonces W1 + W2 = W1 .

Solución.

a) 1) Notemos que 0V ∈ W1 y 0V ∈ W2 , al ser W1 y W2 subespacios. Luego, 0V = 0V + 0V ∈ W1 + W2 ,


lo que implica que W1 + W2 6= ∅.
2) Sean u, v ∈ W1 + W2 y sea α ∈ K. Vamos a mostrar que αu + v ∈ W1 + W2 . Por hipótesis,
existen u1 , v1 ∈ W1 y u2 , v2 ∈ W2 tales que

u = u1 + u2 y v = v1 + v2 .

Entonces,
αu + v = α(u1 + u2 ) + (v1 + v2 ) = (αu1 + v1 ) + (αu2 + v2 ),

y dado que W1 y W2 son subespacios tenemos que αu1 + v1 ∈ W1 y αu2 + v2 ∈ W2 . Por tanto,
αu + v ∈ W1 + W2 .
Por 1) y 2), concluimos que W1 + W2 es un subespacio.
b) Sea w1 ∈ W1 , por ser W2 un subespacio, 0V ∈ W2 , lo que implica que

w1 = w1 + 0V ∈ W1 + W2 .

Por lo tanto, W1 ⊆ W1 + W2 . De forma similar se muestra que W2 ⊆ W1 + W2 .


c) Vamos a mostrar que W1 + W2 ⊆ gen(W1 , W2 ) y que gen(W1 , W2 ) ⊆ W1 + W2 , lo que nos permitirá
concluir que W1 + W2 = gen(W1 , W2 ).
Sea x ∈ W1 + W2 , entonces existen x1 ∈ W1 y x2 ∈ W2 tales que x = x1 + x2 , además, notemos que
x = 1x1 + 1x2 , lo que implica que x es una combinación lineal de elementos en W1 ∪ W2 , es decir,
x ∈ gen(W1 , W2 ), por lo tanto, W1 + W2 ⊆ gen(W1 , W2 ).
Ahora, dado que en los literales a y b hemos mostrado que W1 + W2 es un subespacio que contiene
tanto a W1 como a W2 , entonces
W1 ∪ W2 ⊆ W1 + W2

lo que implica que gen({W1 ∪ W2 }) ⊆ W1 + W2 .


d) Sabemos que W2 ⊆ W1 + W2 . Ahora, sea w ∈ W1 + W2 , queremos probar que

w ∈ W2 .

Para ello, sabemos que


w = w1 + w2 ,

donde w1 ∈ W1 y w2 ∈ W2 . Además, como W1 ⊆ W2 , se tiene que w1 ∈ W2 , con lo cual w ∈ W2 ,


que es lo que se quería demostrar.

1
2. Sean ( E, +, ·, R ) un espacio vectorial, S ⊆ V y α ∈ R. Se define

αS = {αx : x ∈ S}.

Si W es un subespacio vectorial de E, demuestre o refute los siguientes enunciados:

a) W + W = 2W
b) 2W + 2W = W
c) 2W − 2W = {0}, con W 6= {0}

Solución.

a) Se tiene que
W + W = { w1 + w2 ∈ E : w1 ∈ W y w2 ∈ W }

Ya que w1 , w2 ∈ W y por propiedades del espacio vectorial, w1 + w2 ∈ W. Entonces, W + W = W.


Luego,
2W = {2w : w ∈ W }

y como, por propiedades del espacio vectorial, 2w ∈ W, entonces 2W = W; es decir,

W + W = W = 2W

b) Se tiene que
2W + 2W = {2w1 + 2w2 ∈ E : w1 ∈ W y w2 ∈ W }

Ya que w1 ∈ W, entonces por propiedades del espacio vectorial 2w1 ∈ W; similarmente, como
w2 ∈ W, entonces 2w2 ∈ W. Con lo antes dicho, se concluye que 2w1 + 2w2 ∈ W; y, por lo tanto,
2W + 2W = W.
c) Se tiene que

2W − 2W = 2W + (−2)W = {2w1 + (−2)w2 ∈ E : w1 ∈ W y w2 ∈ W }

Ya que w1 ∈ W, entonces por propiedades del espacio vectorial 2w1 ∈ W; similarmente, como w2 ∈
W, entonces (−2)w2 ∈ W. Con lo antes dicho, se concluye que 2w1 + (−2)w2 = 2w1 − 2w2 ∈ W; y,
por lo tanto, 2W − 2W = W, con lo cual se refuta el enunciado.

3. Sean ( E, +, ·, K ) un espacio vectorial y W1 , W2 ⊆ E dos subespacios vectoriales de E de dimensión finita.


Demuestre que W1 + W2 = W2 + W1

Solución.
Sabemos que W1 + W2 = gen( B1 ∪ B2). Además, como B1 ∪ B2 = B2 ∪ B1, entonces

W1 + W2 = gen( B2 ∪ B1),

por lo tanto
W1 + W2 = W2 + W1 .

4. Sean W1 , W2 dos subespacios vectoriales de R4 y {e1 , e2 , e3 , e4 } la base canónica de R4 . Sea

W1 = gen({e1 + e2 , e3 − e4 }) y W2 = gen({−e2 + e1 , e3 + e4 }).

Determinar W1 + W2 y su dimensión.

Solución.

2
Tenemos que W1 = gen ({(1, 1, 0, 0), (0, 0, 1, −1)}) y que W2 = gen ({(1, −1, 0, 0), (0, 0, 1, 1)}), siendo
B1 = {(1, 1, 0, 0), (0, 0, 1, −1)} y que B2 = {(1, −1, 0, 0), (0, 0, 1, 1)} una base de W1 y de W2 , respectiva-
mente.
Calculamos W1 + W2 = gen( B1 ∪ B2 ) = gen ({(1, 1, 0, 0), (0, 0, 1, −1), (1, −1, 0, 0), (0, 0, 1, 1)}). Para
hallar gen( B1 ∪ B2 ), tomemos ( x1 , x2 , x3 , x4 ) ∈ R4 , tenemos que ( x1 , x2 , x3 , x4 ) ∈ gen( B1 ∪ B2 ) si y solo si
existen α1 , α2 , α3 , α4 ∈ R tales que

α1 (1, 1, 0, 0) + α2 (0, 0, 1, −1) + α3 (1, −1, 0, 0) + α4 (0, 0, 1, 1) = ( x1 , x2 , x3 , x4 )

lo cual nos lleva equivalentemente a un sistema de ecuaciones lineales cuya matriz ampliada verifica la
siguiente equivalencia
   
1 0 1 0 | x1 1 0 1 0 | x1
1 0 −1 0 | x  0 1 0 1 | x3 
2
∼ .
  
0 1 0 1 | x3  0 0 −2 0 | x2 − x1 

0 −1 0 1 | x4 0 0 0 2 | x3 + x4

Por lo tanto W1 + W2 = R4 , de donde concluimos que dim(W1 + W2 ) = dim R4 = 4.




5. En R3 sean los subespacios W1 = ( x1 , x2 , x3 ) ∈ R3 : x2 = 0 y W2 = gen({(1, 1, 1), (1, α, 3)}), donde α




es un número real.

a) Calcule la dimensión de W1 , W2 , W1 + W2 y W1 ∩ W2 en función de α.


b) Si α = 1, ¿existe β ∈ R tal que ( β, 2, 1) ∈ W1 + W2 ?

6. Sean W1 , W2 , W3 , subespacios vectoriales del espacio vectorial E de dimensión finita, tal que

a) W1 ⊆ W2 ,
b) W1 + W3 = W2 + W3 , y
c) W1 ∩ W3 = W2 ∩ W3 .

Demostrar que W1 = W2 .

Solución.
Se tiene que
dim(W1 + W3 ) = dim(W1 ) + dim(W3 ) − dim(W1 ∩ W3 ), (1)

y
dim(W2 + W3 ) = dim(W2 ) + dim(W3 ) − dim(W2 ∩ W3 ). (2)

Como W1 + W3 = W2 + W3 , entonces

dim(W1 + W3 ) = dim(W2 + W3 ) (3)

Similarmente, como W1 ∩ W3 = W2 ∩ W3 , entonces

dim(W1 ∩ W3 ) = dim(W2 ∩ W3 ). (4)

(3) aplicado a (1) y (2), junto con (4) resulta en

dim(W1 ) + dim(W3 ) = dim(W2 ) + dim(W3 )

de donde,
dim(W1 ) = dim(W2 ).

Como W1 ⊆ W2 , entonces
W1 = W2 ,

que es lo que se quería demostrar.

3
7. Suponga que W1 y W2 son subespacios de dimensión igual a 4 de un espacio vectorial V con dim(V ) = 6.
Hallar todas las dimensiones posibles de W1 ∩ W2 .

Solución.
Notemos que dim(W1 ∩ W2 ) ≤ dim(V ) = 6. Además, por las dimensiones de W1 y de W2 tenemos
que dim(W1 ∩ W2 ) ≤ 4. Ahora, si se tiene que W1 + W2 = V, entonces dim(W1 + W2 ) = 6, y de

dim(W1 + W2 ) = dim(W1 ) + dim(W2 ) − dim(W1 ∩ W2 ),

concluimos que dim(W1 ∩ W2 ) ≥ 2. Por lo tanto, 2 ≤ dim(W1 ∩ W2 ) ≤ 4.

8. En los siguientes literales se dan un espacio vectorial V y dos subespacios W1 y W2 . En cada caso,
determinar W1 + W2 y estudiar si W1 y W2 están o no en suma directa de V.

a) V = R3 , W1 = { x ∈ V : x1 + x2 = 0, x1 + x3 = 0} y W2 = { x ∈ V : x1 = 0, x2 = 0}.
( )
4
b) V = R4 , W1 = gen({(1, −1, 0, 0), (1, 0, −1, 0), (1, 0, 0, −1)}) y W2 = x∈V: ∑ xk = 0 .
k =1

c) V = R4 , W1 = { x ∈ V : x2 + x3 + x4 = 0} y W2 = { x ∈ V : x1 + x2 = 0, x3 = 2x4 }.
d) V = R n×n , W1 = A ∈ V : A = A T y W2 = A ∈ V : A = − A T .
 
( ! ) ( ! )
a b a b
e) V = R2×2 , W1 = : b = − a ∈ R y W2 = : a, b, c ∈ R .
c d c −a
!
1
f) V = R 3×2 , W1 = { A ∈ V : AM = 0} y W2 = { A ∈ V : N ⊺ A⊺ − AN = 0}, siendo M =
−3
!
1 0 0
yN= .
0 −1 1
g) V = R2 [t], W1 = { p ∈ V : p′ (0) = p(0)} y W2 = R1 [t].
h) V = R3 [t], W1 = { p ∈ V : p(0) + p′ (0) = 0} y W2 = { p ∈ V : p′′ (0) − p′ (0) = 0}.
i) V es el espacio de funciones reales, W1 = { f ∈ V : f (− x ) = f ( x ), para todo x ∈ R } y
W2 = { f ∈ V : f (− x ) = − f ( x ), para todo x ∈ R }.

Solución.

a) Verifique que una base de W1 es B1 = {(1, −1, −1)} y que una base de W2 es B2 = {(0, 0, 1)}. Luego,
W1 + W2 = gen( B1 , B2 ) = {( x1 , x2 , x3 ) ∈ V : x1 + x2 = 0}. Por lo tanto, al ser W1 + W2 6= R3 , se
concluye que W1 y W2 no están en suma directa de V.
b) Verifique que W1 + W2 = W1 y que, por tanto, W1 y W2 no están en suma directa de V.
c) Verifique que W1 + W2 = R4 y que W1 ∩ W2 6= {0}, por lo cual, W1 y W2 no están en suma directa
de V.
d) Sea A ∈ R n×n , tenemos que
1 1
A= ( A + AT ) + ( A − AT )
2 2
1 1
donde ( A + A T ) ∈ W1 y ( A − A T ) ∈ W2 , luego, W1 + W2 = R n×n .
2 2
Supongamos que W1 ∩ W2 6= ∅, existe B ∈ W1 ∩ W2 , luego, B ∈ W1 y B ∈ W2 , lo que implica que

BT = B y B T = − B,

de lo cual, B = − B, es decir, 2B = 0, esto es B = 0. Luego, W1 ∩ W2 = {0}.

4
Ya que hemos mostrado que W1 + W2 = R n×n y que W1 ∩ W2 = {0}, concluimos que

W1 ⊕ W2 = R n×n ,

es decir, W1 y W2 están en suma directa de V.


Otra forma de calcular la intersección entre W1 y W2 es

W1 ∩ W2 = { A ∈ V : A = A T , A = − A T }.
( ! ! !)
1 −1 0 0 0 0
e) Notemos que una base de W1 es B1 = , , y que una base de W2 es
0 0 1 0 0 1
( ! ! !)
1 0 0 1 0 0
B2 = , , , verifique que W1 + W2 = R2×2 .
0 −1 0 0 1 0
Ahora, dado que dim(W1 + W2 ) = 4 y que dim(W1 ) = dim(W2 ) = 3, concluimos que

dim(W1 ∩ W2 ) = 2,

lo que implica que W1 ∩ W2 6= {0}, por lo tanto, W1 y W2 no están en suma directa de V.


  
 a b
 

f ) Verifique que W1 + W2 =  c d  : c + 3d + e + 3 f = 0 , y que W1 y W2 no están en suma
 

 e f 

directa de V.
g) Verifique que W1 + W2 = R2 [t], y que W1 y W2 no están en suma directa de V.
h) Verifique que W1 + W2 = R3 [t], y que W1 y W2 no están en suma directa de V.
i) Verifique que W1 ⊕ W2 = V, es decir, W1 y W2 están en suma directa de V.
Para esto, considere a h, g : R → R definidas por
f ( x ) + f (− x ) f ( x ) − f (− x )
h( x ) = y g( x ) =
2 2
para todo x ∈ R. Verifique que h ∈ W1 , que g ∈ W2 y que f = h + g. Además,

W1 ∩ W2 = { f ∈ V : f (− x ) = f ( x ), f (− x ) = − f (− x ), para todo x ∈ R } .

9. Sea W = { p( x ) ∈ R4 [ x ] : p′′ (−1) = 0}

a) Encontrar una base para W


b) Completar la base del literal anterior a una base para R4 [ x ]
c) Determinar un subespacio U de R4 [ x ] tal que R4 [ x ] = U ⊕ W

Solución.

a) Tenemos
W = { ax4 + bx3 + cx2 + dx + e ∈ R4 [ x ] : 12a − 6b + 2c = 0}.

Sea ax4 + bx3 + cx2 + dx + e ∈ R4 [ x ], entonces ax4 + bx3 + cx2 + dx + e ∈ W si y solo si

12a − 6b + 2c = 0,

de donde se obtiene que


1 1
a= b − c,
2 6
por lo tanto
 
4 3 2 1 1
ax + bx + cx + dx + e = b − c x4 + bx3 + cx2 + dx + e
2 6

5
 
1 4 1
=b x + x3 + c(− x4 + x2 ) + d( x ) + e(1).
2 6
Con esto, se obtiene que  
1 4 1
B= x + x3 , − x4 + x2 , x, 1
2 6
es un conjunto generador de E.
Ahora, tomamos α1 , α2 , α3 , α4 ∈ R y escribimos la combinación lineal nula con los elementos de B,
   
1 4 1 4
α1 x + x + α2 − x + x + α3 ( x ) + α4 (1) = 0x4 + 0x3 + 0x2 + 0x + 0.
3 2
2 6
El resultado anterior conduce al sistema
1 1
2 α1 − 6 α2 = 0,
α1 = 0,
α2 = 0,
α3 = 0,
α4 = 0,
de donde, notamos que α1 = α2 = α3 = α4 = 0. Con lo cual, se tiene que B es un conjunto
linealmente independiente, y, por tanto, una base de W.
b) Notamos que
card( B) = 4 = dim(W ).

Como dim (R4 [ x ]) = 5, debemos completar B con un vector q( x ) ∈ R4 [ x ] tal que

{q( x )} ∪ B
sea un conjunto linealmente independiente. Para ello, debemos determinar q( x ) tal que

q( x ) ∈
/ gen( B) = W.

Entonces tomemos q( x ) = x3 + x, y verifique que q( x ) ∈ / W y que {q( x )} ∪ B es linealmente


independiente. Ahora, en virtud de lo anterior, concluya que
 
′ 1 4 3 1 4 2 3
B = x + x , − x + x , x, 1, x + x
2 6
es una base de R4 [ x ].
c) Del literal anterior tenemos
span( B′ ) = R4 [ x ],

de donde
B′ = B ∪ {q( x )} ,

con q( x ) = x3 + x. Además, sabemos que

gen( B) = W.

Denotamos gen({q( x )}) = U, notemos que dim(U ) = 1. Luego, sabiendo que

dim(U + W ) = dim(U ) + dim(W ) − dim(U ∩ W ),

se tiene que

dim(W ∩ U ) = dim(U ) + dim(W ) − dim(U + W ) = 1 + 4 − 5 = 0

entonces,
U ⊕ W = R 4 [ x ].

6
10. En R1 [t], se define el subespacio vectorial

V = { p(t) ∈ R1 [t] : p′ (0) = p(1)}.

Determinar un subespacio vectorial W de R1 [t] tal que

R1 [t] = V ⊕ W.

Solución.
Se tiene que:

V = { p(t) = a + bt : p′ (0) = p(1)}


= { a + bt : b = a + b}
= {bt : b ∈ R }
= h{t}i
donde BV = {t} es base de V.
Luego, tomando como referencia la base canónica de R1 [t] tomamos

W = h{1}i

donde W es un subespacio vectorial, puesto que la cápsula de un conjunto genera un subespacio vecto-
rial, además
BW = {1}

es base de W.
Se tiene que:
V + W = h{ Bv ∪ BW }i = h{1, t}i = R1 [t]

además, como dim(V ∩ W ) = 0, tenemos que

V ∩ W = { 0v }

entonces se tiene que


R1 [t] = V ⊕ W.

11. Decidir si las siguientes afirmaciones son verdaderas o falsas, justificando su respuesta.

a) Si W1 , W2 son subespacios de R3 con dim(W1 ) = dim(W2 ) = 2, entonces existe v ∈ R3 r {0} tal


que v ∈ W1 ∩ W2 .
b) Si W1 , W2 y W3 son subespacios de R11 con dim(W1 ) = dim(W2 ) = dim(W3 ) = 4, entonces
dim(W1 ∩ W2 ∩ W3 ) ≥ 1.

Solución.

a) Notemos que solo existen dos opciones a ser estudiadas: W1 = W2 y W1 6= W2 , que en ambos casos
nos lleva a concluir que W1 ∩ W2 6= {0}, lo que implica que existe al menos un v ∈ R3 r {0} tal que
v ∈ W1 ∩ W2 . Por lo tanto, el enunciado es verdadero.
b) El enunciado es falso, para mostrarlo consideremos la base canónica de R11 y definamos

W1 = W2 = gen ({e1 , e2 , e3 , e4 }) y W3 = gen ({e5 , e6 , e7 , e8 }) .

De donde, dim(W1 ) = dim(W2 ) = dim(W3 ) = 4 con

W1 ∩ W2 = W1 + W2 = W1 y W1 ∩ W3 = W2 ∩ W3 = {0} ,

7
lo que nos permite concluir que

W1 + W2 + W3 = { x ∈ R11 : x9 = x10 = x11 = 0} y W1 ∩ W2 ∩ W3 = {0}

y que dim(W1 ∩ W2 ∩ W3 ) = 0.

Ejercicios sugeridos: 1a, 1b, 1c, 4, 6, 7, 8a, 8d, 8e y 10.

También podría gustarte